Administración     

Olimpiadas de Matemáticas
Página de preparación y problemas

Selector
La base de datos contiene 1154 problemas y 775 soluciones.
OME Local
OME Nacional
OIM
OME Andalucía
Retos UJA
+20
Problema 150
Hallar todas las sucesiones $0\lt a_1\leq a_2\leq\ldots\leq a_n$ de numeros reales verificando \[\left\{\begin{array}{rcl} a_1+a_2+\ldots+a_n&=&96\\a_1^2+a_2^2+\ldots+a_n^2&=&144\\a_1^3+a_2^3+\ldots+a_n^3&=&216 \end{array}\right.\]
pistasolución 1info
Pista. Intenta utilizar la desigualdad de Cauchy-Schwarz involucrando a las cantidades del enunciado.
Solución. La desigualdad de Cauchy-Schwarz aplicada a los vectores \begin{eqnarray*} u&=&(a_1^{3/2},a_2^{3/2},\ldots,a_n^{3/2}),\\ v&=&(a_1^{1/2},a_2^{1/2},\ldots,a_n^{1/2}), \end{eqnarray*} nos dice que \[(a_1^2+a_2^2+\ldots+a_n^2)^2\leq(a_1^3+a_2^3+\ldots+a_n^3)(a_1+a_2\ldots+a_n).\] Observemos que en nuestro caso se da la igualdad puesto que $144^2=96\cdot216$ luego $u$ y $v$ son proporcionales, es decir, existe $\lambda\in\mathbb{R}$ tal que $u=\lambda v$, luego $a_k^{1/2}=\lambda a_k^{3/2}$ para $1\leq k\leq n$. Elevando al cuadrado, $a_k^3=\lambda^2a_k$ y, sumando en $k$, ha de cumplirse que $216=96\lambda^2$, de donde $\lambda=\frac{3}{2}$. Por tanto, $a_k^3=\frac{9}{4}a_k$ de donde tiene que ser $a_k=\frac{3}{2}$ para todo $k$ y, finalmente, tenemos que $n=64$. Es fácil ver que esta sucesión constante verifica las tres ecuaciones del enunciado.
Si crees que el enunciado contiene un error o imprecisión o bien crees que la información sobre la procedencia del problema es incorrecta, puedes notificarlo usando los siguientes botones:
Informar de error en enunciado Informar de procedencia del problema
Problema 149
Dado $n\geq 1$, hallar todas las $n$-uplas de números reales $x_1,x_2,\ldots,x_n\geq 1$ tales que \[\sqrt{x_1}+\sqrt[3]{x_2}+\ldots+\sqrt[n+1]{x_n}=\sqrt{n}\sqrt{x_1+x_2+\ldots+x_n}.\]
pistasolución 1info
Pista. Realmente, para cualesquiera $x_1,x_2,\ldots,x_n\geq 1$, una de las desigualdades en la igualdad del enunciado se cumple, luego el problema puede reducirse a estudiar cuándo se alcanza la igualdad en una desigualdad. La desigualdad de Cauchy-Schwarz puede ser útil para terminar el problema.
Solución. Evidentemente, si tomamos $x_1=x_2=\ldots=x_n=1$, tenemos una solución. Veremos ahora que, independientemente de los valores de las variables siempre se tiene una desigualdad $\leq$ en la expresión del enunciado y veremos que la igualdad sólo se alcanza para esta solución.

Aplicando la desigualdad de Cauchy-Schwarz a los vectores \begin{eqnarray*} u&=&\left(\sqrt{x_1},\sqrt[3]{x_2},\ldots,\sqrt[n+1]{x_n}\right)\\ v&=&\left(1,1,\ldots,1\right) \end{eqnarray*} llegamos a que \[\sqrt{x_1}+\sqrt[3]{x_2}+\ldots+\sqrt[n+1]{x_n}\leq\sqrt{n}\sqrt{x_1+x_2^{2/3}+x_3^{2/4}+\ldots+x_n^{2/(n+1)}}\] Ahora bien, como $x_2,\ldots,x_n$ son mayores o iguales que uno que uno, se tiene que $x_k^{2/(k+1)}\leq x_k$ pues $\frac{2}{k+1}\leq 1$. Esto demuestra la desigualdad que queremos y, si la igualdad se alcanza, entonces $x_2=\ldots=x_n=1$ por la última desigualdad y $x_1=1$ por la igualdad en la desigualdad de Cauchy-Schwarz. Deducimos que $x_1=x_2=\ldots=x_n=1$ es la única solución al problema.

Si crees que el enunciado contiene un error o imprecisión o bien crees que la información sobre la procedencia del problema es incorrecta, puedes notificarlo usando los siguientes botones:
Informar de error en enunciado Informar de procedencia del problema
Problema 148
Dados tres números reales positivos $x,y,z\geq 0$ tales que $x+y+z=1$, demostrar que \[\left(1+\frac{1}{x}\right)\left(1+\frac{1}{y}\right)\left(1+\frac{1}{z}\right)\geq 64.\] Analizar para qué valores de $x,y,z$ se obtiene la igualdad.
pistasolución 1info
Pista. Manipula usando la igualdad $x+y+z=1$ y utiliza la desigualdad entre las medias aritmética y geométrica.
Solución. Cambiando el $1$ de los numeradores de las fracciones por $x+y+z$ y operando, la desigualdad a probar es equivalente a \[(2x+y+z)(x+2y+z)(x+y+2z)\geq 64xyz.\] Ahora bien, usando repetidamente la desigualdad entre las medias aritmética y geométrica, tenemos que \begin{eqnarray*} 64xyz=64\sqrt{xy}\sqrt{yz}\sqrt{xz}&\leq& 8(x+y)(y+z)(x+z)\\ &=&8\sqrt{(x+y)(y+z)}\sqrt{(x+y)(x+z)}\sqrt{(x+z)(y+z)}\\ &\leq&(x+y+y+z)(x+y+x+z)(x+z+y+z)\\ &=&(2x+y+z)(x+2y+z)(x+y+2z) \end{eqnarray*} con lo que la desigualdad está probada. La igualdad se alcanza si, y sólo si, $x=y=z=\frac{1}{3}$.
Si crees que el enunciado contiene un error o imprecisión o bien crees que la información sobre la procedencia del problema es incorrecta, puedes notificarlo usando los siguientes botones:
Informar de error en enunciado Informar de procedencia del problema
Problema 105
Supongamos que $a,b,c,d\in\mathbb{R}$ son números positivos que satisfacen la condición $ab+bc+cd+da=1$. Demostrar que \[\frac{a^3}{b+c+d}+\frac{b^3}{a+c+d}+\frac{c^3}{a+b+d}+\frac{d^3}{a+b+c}\geq\frac{1}{3}\] y analizar en qué casos se alcanza la igualdad.
pistasolución 1info
Pista. Utilizar la desigualdad de Chebyshev para transformar la desigualdad del enunciado.
Solución. Supondremos sin perder generalidad que $a\geq b\geq c\geq d\gt 0$ y llamaremos $E$ al miembro de la izquierda de la desigualdad del enunciado. Si definimos \[x_1=a^3,\quad x_2=b^3,\quad x_3=c^3,\quad x_4=d^3,\] \[y_1=\frac{1}{b+c+d},\quad y_2=\frac{1}{a+c+d},\quad y_3=\frac{1}{a+b+d},\quad y_4=\frac{1}{a+b+c},\] se cumple que $x_1\geq x_2\geq x_3\geq x_4\gt 0$ e $y_1\geq y_2\geq y_3\geq y_4\gt 0$. Por tanto, podemos aplicar la desigualdad de Chebyshev a estos números y obtenemos que \[E\geq\frac{1}{4}(a^3+b^3+c^3+d^3)\left(\frac{1}{b+c+d}+\frac{1}{a+c+d}+\frac{1}{a+b+d}+\frac{1}{a+b+c}\right).\] La desigualdad entre las medias cúbica y aritmética aplicada a los $x_i$ nos dice que \[\frac{1}{4}(a^3+b^3+c^3+d^3)\geq\frac{1}{64}(a+b+c+d)^3\] y la desigualdad entre las medias aritmética y armónica aplicada a los $y_i$ que \[\frac{1}{b+c+d}+\frac{1}{a+c+d}+\frac{1}{a+b+d}+\frac{1}{a+b+c}\geq\frac{16}{3(a+b+c+d)}.\] Usando estas dos últimas desigualdades, llegamos a que \[E\geq\frac{16(a+b+c+d)^3}{3\cdot 64(a+b+c+d)}=\frac{1}{12}(a+b+c+d)^2.\] Finalmente, usando que la condición del enunciado se escribe como $(a+c)(b+d)=1$ y usando la desigualdad entre las medias aritmética y geométrica, obtenemos \[(a+b+c+d)^2\geq 4(a+c)(b+d)=4,\] con lo que $E\geq\frac{1}{3}$ como queríamos probar. Si la igualdad se alcanza, entonces en la desigualdad entre las medias cúbica y aritmética para los $x_i$ se deduce que $a=b=c=d$ y , por la condición $ab+bc+cd+da=1$, estos cuatro números tienen que ser iguales a $\frac{1}{2}$, Se comprueba que, para esa elección, se alcanza la igualdad luego esa es la única solución.
Si crees que el enunciado contiene un error o imprecisión o bien crees que la información sobre la procedencia del problema es incorrecta, puedes notificarlo usando los siguientes botones:
Informar de error en enunciado Informar de procedencia del problema
Problema 37
Demostrar la siguiente desigualdad para cualquier número natural \(n\) \[\frac{1}{2\sqrt{n}}\lt\frac{1}{2}\cdot\frac{3}{4}\cdot\frac{5}{6}\cdots\frac{2n-1}{2n}\lt\frac{1}{\sqrt{2n+1}}\]
pistasolución 1info
Pista. Eleva la desigualdad al cuadrado. ¿Cómo se puede transformar el producto del enunciado para simplificar los numeradores y denominadores?
Solución. Llamemos \(E\) al producto que aparece en la desigualdad. Usando que \(\frac{k-1}{k}\lt\frac{k}{k+1}\lt\frac{k+1}{k+2}\) para cualquier número natural \(k\), se tiene que \[\frac{k-1}{k}\frac{k}{k+1}\lt\left(\frac{k}{k+1}\right)^2\lt\frac{k}{k+1}\frac{k+1}{k+2}\] Usando esto para acotar \(E^2\), tenemos por un lado que \[E^2\lt\frac{1}{2}\frac{2}{3}\frac{3}{4}\frac{4}{5}\cdots\frac{2n-1}{2n}\frac{2n}{2n+1}=\frac{1}{2n+1}\] y, por otro lado, \[E^2\gt\frac{1}{2}\frac{1}{2}\frac{2}{3}\frac{3}{4}\cdots\frac{2n-2}{2n-1}\frac{2n-1}{2n}=\frac{1}{4n}\] De estas dos desigualdades se deducen las del enunciado.
Si crees que el enunciado contiene un error o imprecisión o bien crees que la información sobre la procedencia del problema es incorrecta, puedes notificarlo usando los siguientes botones:
Informar de error en enunciado Informar de procedencia del problema
José Miguel Manzano © 2010-2024. Esta página ha sido creada mediante software libre